lauteri2
Thanks Received: 0
Forum Guests
 
Posts: 5
Joined: November 06th, 2009
 
 
 

Logic Challenge #23 - Old Blue Eyes

by lauteri2 Mon Mar 22, 2010 6:00 pm

1. a
2. c
3. b
4. d
5. a
6. c
7. a

-Gina :)
 
mybiztalk
Thanks Received: 0
Forum Guests
 
Posts: 2
Joined: July 09th, 2009
 
 
 

Re: Fortnightly Logic Challenge #23 - Old Blue Eyes

by mybiztalk Mon Mar 22, 2010 6:03 pm

1)A
2)C
3)E
4)E
5)D
6)C
7)A

Ari :idea:
Last edited by mybiztalk on Mon Mar 22, 2010 6:29 pm, edited 5 times in total.
 
daria_simionescu
Thanks Received: 0
Forum Guests
 
Posts: 1
Joined: February 08th, 2010
 
 
 

Re: Fortnightly Logic Challenge #23 - Old Blue Eyes

by daria_simionescu Mon Mar 22, 2010 6:07 pm

1) a
2)C
3)E
4)E
5)C
6)C
7)A
 
lauteri2
Thanks Received: 0
Forum Guests
 
Posts: 5
Joined: November 06th, 2009
 
 
 

Re: Fortnightly Logic Challenge #23 - Old Blue Eyes

by lauteri2 Mon Mar 22, 2010 6:20 pm

Actually, #3 is E. There is no possible way that could work out. If H is in third place, then either L is in fourth or second. If L is in fourth, that means O is in fifth and then J would be in second, which leaves no M/P_P/M slot. If L is before H, then that means O is first, L is second, and H is third, leaving J fourth, which violates the rule that H has to come after J.
 
lauteri2
Thanks Received: 0
Forum Guests
 
Posts: 5
Joined: November 06th, 2009
 
 
 

Re: Fortnightly Logic Challenge #23 - Old Blue Eyes

by lauteri2 Mon Mar 22, 2010 6:31 pm

1. Which one of the following could be the order of auditions, listed from first to last?

(A) Jordan, Lewin, Hiroshi, Olin, Marin, Kirkwood, Pan
(B) Jordan, Hiroshi, Lewin, Olin, Marin, Pan, Kirkwood
(C) Olin, Hiroshi, Lewin, Jordan, Pan, Kirkwood, Marin
(D) Pan, Olin, Marin, Jordan, Kirkwood, Hiroshi, Lewin
(E) Marin, Jordan, Pan, Lewin, Olin, Hiroshi, Kirkwood

a, These ones are the easiest, just go through with each of the rules and once you find one that doesn't work, cross it off (and move on to the next rule, I've never or at least hardly ever seen a rule that removed two answer choices).

2. If Olin auditions first, which one of the following must be true?

(A) Marin auditions third.
(B) Pan auditions third.
(C) Kirkwood auditions second.
(D) Jordan auditions third.
(E) Hiroshi auditions seventh.

c, Here we know that if Olin is first, then according to the rules J must be fourth. Now, the next major block is the M/P block. Because of the placement of O/J, M/P can either go in slots 3/5 or 5/7. However, we have to choose slots 3/5 or else H and L, which have to be next to each other, can't be together. Therefore, we know either M and P (not in that exact order) make up slots 3 and 5 and because H (and therefore, L) have to come after J, we know they are in slots 6 and 7. Therefore, K is forced to audition second.

3. If Lewin auditions immediately before Olin, each of the following could be true EXCEPT:

(A) Kirkwood auditions second.
(B) Marin auditions seventh.
(C) Pan auditions third.
(D) Jordan auditions first.
(E) Hiroshi auditions third.


Here another condition is added, that there is an LO block. I explained the answer up above.
 
ryan.carter
Thanks Received: 0
Vinny Gambini
Vinny Gambini
 
Posts: 1
Joined: December 03rd, 2009
 
 
 

Re: Fortnightly Logic Challenge #23 - Old Blue Eyes

by ryan.carter Mon Mar 22, 2010 6:36 pm

1. A
2. C
3. E
4. E
5. C
6. C
7. A
 
lauteri2
Thanks Received: 0
Forum Guests
 
Posts: 5
Joined: November 06th, 2009
 
 
 

Re: Fortnightly Logic Challenge #23 - Old Blue Eyes

by lauteri2 Mon Mar 22, 2010 6:42 pm

5. actually c...I incorrectly switched the letters when I was doing a..
 
lauteri2
Thanks Received: 0
Forum Guests
 
Posts: 5
Joined: November 06th, 2009
 
 
 

Re: Fortnightly Logic Challenge #23 - Old Blue Eyes

by lauteri2 Mon Mar 22, 2010 6:46 pm

Yeah, Ryan Carter has it.
 
perfectparadise1
Thanks Received: 0
Forum Guests
 
Posts: 10
Joined: December 28th, 2009
 
 
 

Re: Fortnightly Logic Challenge #23 - Old Blue Eyes

by perfectparadise1 Tue Mar 23, 2010 12:11 am

YES THIS IS AN EDIT! $#%$!!! I was mistaken on #5. The correction is beside it in red. I suck. :mrgreen:

Atlas LSAT Logic Game Challenge #23

Old Blue Eyes

1. Which one of the following could be the order of auditions, listed from first to last?

(A) Jordan, Lewin, Hiroshi, Olin, Marin, Kirkwood, Pan
(B) Jordan, Hiroshi, Lewin, Olin, Marin, Pan, Kirkwood
(C) Olin, Hiroshi, Lewin, Jordan, Pan, Kirkwood, Marin
(D) Pan, Olin, Marin, Jordan, Kirkwood, Hiroshi, Lewin
(E) Marin, Jordan, Pan, Lewin, Olin, Hiroshi, Kirkwood

Going through the rules:

A. Satisfies all the rules so this is it (I'd write this out in a diagram as well for reference since it is a correct ordering and may prove useful in later questions)
~B. Exactly one audition between Marin and Pan this choice has them consecutively placed
~C. Jordan comes before Hiroshi so C is out
~ D. Jordan and Olin need to have TWO auditions between them not just one
~E. Hiroshi and Lewin have to be consecutive in either order so E is out

2. If Olin auditions first, which one of the following must be true?

(A) Marin auditions third.
(B) Pan auditions third.
(C) Kirkwood auditions second.
(D) Jordan auditions third.
(E) Hiroshi auditions seventh.

Olin goes first so we are faced with a limited possibilities scenario as referenced in the initial deductions:

1 2 3 4 5 6 7
O K M/P J M/P H/L H/L
O H/L L/H J M/P K P/M

Since Olin is going First and not Jordan then the second scenario doesn't work since it precludes Jordan from meeting the requirement that he go before Hiroshi.

If O goes first J must go 4th. For the M_P/P_M requirement to work M and P MUST go 3 and 5 though in either order. Since H and L MUST be consecutive then they are forced into the 6 and 7 spots.

Looking at the answer choices only C works. (C) Kirkwood auditions second.

Note that A B and E can be true but don't have to be. Choice D, Jordan going third is an impossibility as noted in the initial deductions since it would give no room for the M/P_P/M block.

A shortcut to this would be to realize that M and P are interchangeable as are H and L (as long as Jordan is before H of course) so if M AND P are among answer choices for the same position then logically neither can be a MUST.

3. If Lewin auditions immediately before Olin, each of the following could be true EXCEPT:

(A) Kirkwood auditions second
B) Marin auditions seventh.
(C) Pan auditions third.
(D) Jordan auditions first.
(E) Hiroshi auditions third.

So we have a new block LO. H and L still have to be consecutive so we can make an HLO block. Now we have the J/O _ _ O/J rule and the J comes before H rules so we can deduce that JHLO is a block. This means that K has to come between M and P. And we are left with the below options. Only choice E can't be true, (E) Hiroshi auditions third cannot be true and is the right answer.

1 2 3 4 5 6 7
J H L O M/P K P/M
M/P K P/M J H L O

4. If Kirkwood auditions sometime between Jordan and Olin, which one of the following must be true?

(A) Kirkwood auditions second.
(B) Either Marin or Pan auditions fifth.
(C) Either Marin or Pan auditions first.
(D) Either Hiroshi or Lewin auditions last.
(E) Jordan auditions either first or fourth.

In order for K to go between J and O, J and O would have to go 1st and 4th or 2nd and 5th. IF J or O goes first, then :

1 2 3 4 5 6 7
O/J K M/P J/O M/P H/L H/L

This helps us about as much as an IPhone helps a drunk naked wombat at a Luau. Choices A, B, D, and E are still valid.

IF J or O goes second:

1 2 3 4 5 6 7
M/P O/J P/M K J/O H/L L/H

Now we can satisfy the rules and see that choices A, B, and E are invalid. But HOLD on this is a MUST be true question not a MAY be true or CAN be true so where else can the J/O blocks go and have K in between them? Since all the choices aside from D have been eliminated this isn't necessary but for good measure:

Remember, the HL/LH block has to go AFTER J.

J/O in 3 and 6

1 2 3 4 5 6 7
J/O O/J

- This scenario won't allow for the M/P_P/M block and the HL/LH block to fit even with or without the new condition of K being between O and J (this was deduced earlier)

1 2 3 4 5 6 7

J H/L L/H O

- This scenario allows no room for K to fit between J and O

So yup (D) Either Hiroshi or Lewin auditions last.

5. There are exactly two possible orders of auditions if which one of the following is true?

(A) Hiroshi auditions immediately after Jordan.
(B) Kirkwood auditions immediately after Olin.
(C) Lewin auditions immediately after Olin.
(D) Marin auditions immediately after Jordan.
(E) Pan auditions immediately before Olin.

I HATE these types of questions with a passion. FOR GOOD REASON TOO!!!

Choice A yields 4 answers since J could go first yielding JHLOMKP or JHLOPKM or J can go fourth yielding MKPJHLO or PKMJHLO. As someone explained in a later post the answer is C since L coming after O creates limitations as to where J can go since H has to always come after J. Choice A actually has protection for that rule built in by its nature. So MJPKOLH and PJMKOLH become the only options if L follows O. Now I'm going to go and pay a genetic engineer for Einsteins genome so this doesn't happen to me again. And you think I'm kidding about that huh . . .

Okay instead of wasting life (and if it were a test then valuable exam time) take a quick scan of the choices and look for the most limiting conditions or for situations you have in prior arrangements from the other questions. Question 3 involves a set up where H comes immediately after J.

1 2 3 4 5 6 7
J H L O M/P K P/M

This results in only two possible orderings according to the rules thus :

(A) Hiroshi auditions immediately after Jordan allow for exactly two possible orders.

Is it worth looking at the other options? If one is incessantly bored and has no social life whatsoever then sure go all out then go to match.com or a local brothel please for the love of God.

Incidentally, this scenario involves J and H who have the most restrictions of any of the students (along with L). So it's a good bet something further restricting J and L or J and H in the answer will produce the most limiting scenario.

6. If Marin auditions first, which one of the following must be true?

(A) Kirkwood auditions before Pan.
(B) Olin auditions after Pan.
(C) Hiroshi auditions after Kirkwood.
(D) Lewin auditions before Olin.
(E) Jordan auditions after Kirkwood.

If M goes first then P must go third. The J/O _ _ O/J block requires 4 spaces so it can't start later than space 4. Furthermore we know that J must come before H and that H and L must be consecutive so J cannot go 7th. The following scenarios work withins these confines and have Marin going first.

1 2 3 4 5 6 7
M K P J H/L L/H O
M O P K J H/L L/H
M J P K O H/L L/H

(C) Hiroshi auditions after Kirkwood.

Hiroshi has to audition after Kirkwood no matter what.


7. If the condition that Hiroshi’s audition will take place immediately before or immediately after Lewin’s audition is replaced with the condition that exactly one audition takes place between Hiroshi and Lewin, and if all other conditions remain in effect, each of the following could be true
EXCEPT:

(A) Olin auditions fourth.
(B) Jordan auditions third.
(C) Kirkwood auditions first.
(D) Lewin auditions sixth.
(E) Hiroshi auditions fourth.

Rule changing questions are as fun as getting a lobotomy from a quadriplegic.

So now H_L or L_H as well as

1. J/O _ _ O/J
2. M/P _ PM
3. J . . . . H/L

Let's combine rule 3 with this new rule and make the rule:

J . . . H _ L/J . . . L _ H

Now look at the choices. Two of the choices offer a person auditioning 4th. Plug in an work backwards with those.

(A) Olin auditions fourth.
. So J must go in 7 or 1. J can't go in 7 since it has to go before H. So J can only go in 1.

1 2 3 4 5 6 7
J O

Only K can go in 2 since H/L_L/H and M/P_P/M would get in a bloody fight with O over 4th (check out this
Code: Select All Code
http://www.nytimes.com/2010/02/07/world/asia/07karaoke.html
to what happens in the Phillipines when people sing Sinatra and know that I am not kidding about a bloody fight).

1 2 3 4 5 6 7
J K O

There is no way now to satisfy the H/L _ L/H and M/P _ P/M blocks. So A is the correct answer.
The only way for the H/L_L/H and M/P_P/M blocks to work is for the to be all in a row and O in the 4 spots prevents that. So (A) Olin auditions fourth cannot be true and is the right answer.
Last edited by perfectparadise1 on Wed Mar 24, 2010 3:53 am, edited 1 time in total.
 
tad.kim
Thanks Received: 0
Forum Guests
 
Posts: 3
Joined: December 08th, 2009
 
 
 

Re: Fortnightly Logic Challenge #23 - Old Blue Eyes

by tad.kim Tue Mar 23, 2010 1:18 am

1. A
2. C
3. E
4. D
5. D
6. C
7. A
 
ryyang1118
Thanks Received: 1
Forum Guests
 
Posts: 7
Joined: February 24th, 2010
 
 
 

Re: Fortnightly Logic Challenge #23 - Old Blue Eyes

by ryyang1118 Tue Mar 23, 2010 1:47 am

1. A
2. C
3. E
4. D
5. C
6. C
7. A
 
allisongia
Thanks Received: 0
Vinny Gambini
Vinny Gambini
 
Posts: 1
Joined: February 16th, 2010
 
 
 

Re: Fortnightly Logic Challenge #23 - Old Blue Eyes

by allisongia Tue Mar 23, 2010 3:07 am

1. A
2. C
3. E
4. B
5. A
6. C
7. A

Allison :roll:
 
benjaminbbrown
Thanks Received: 0
Forum Guests
 
Posts: 2
Joined: January 09th, 2010
 
 
 

Re: Fortnightly Logic Challenge #23 - Old Blue Eyes

by benjaminbbrown Tue Mar 23, 2010 12:48 pm

1. A
2. C
3. E
4. D
5. E
6. C
7. A
 
tayjones
Thanks Received: 0
Forum Guests
 
Posts: 5
Joined: February 02nd, 2010
 
 
 

Re: Fortnightly Logic Challenge #23 - Old Blue Eyes

by tayjones Wed Mar 24, 2010 1:59 am

I believe @ryyang1118 already got the correct answers. I'll give the explanation a try.

This is a simple line game with seven elements, or "auditions," each used once. The rules are straightforward:

1. J and O are separated by exactly two elements. The rule does not specify which comes first: J/O _ _ O/J
2. M and P are separated by exactly one element, although they may be in any order: M/P _ P/M
3. H is immediately before or immediately after L, making a two letter block: LH/HL
4. We are given a loose ordering rule. J is before H: J-H

Conclusions/deductions:
Combining rules 3 and 4, we see that J cannot be first, nor can it be any later than fifth.
Rule 4 also tells us that H cannot be first. K can be anywhere. That's enough to get started on the questions.

1. "Grab-a-rule." Rule 1 eliminates answer choice (D), 2 gets (B), 3 gets (E), and 4 gets (C). (A) is the remaining answer and is an acceptable order.

2. If O is first, J must be fourth. From our setup we know that the LH/HL block must come after J. Can it be fifth and sixth? No, because that would not leave room for the M/P_P/M block. HL/LH must be sixth and seventh, with M and P on either side of J. K must be second, immediately after O, as described by the correct answer choice, (C). Here is one such configuration: OKMJPLH.
(A) M can be third or fifth.
(B) same analysis as (A). P can be third or fifth.
(C) This is correct. K is the only singer who can be second.
(D) This cannot be true. See question analysis.
(E) H can be seventh or eighth.

3. This question gives us an LO block, which must become HLO to satisfy rule 3. J must be immediately before H because of rules 1 and 4. So now we have a JHLO block taking up more than half the sequence. There are two places we can put this block and still accommodate M/P_P/M: the very beginning or the very end. Looking at the answer choices, the only one that CANNOT be true is (E), which is the correct answer.
(A) K can be second: MKPJHLO
(B) M can be seventh: JHLOPKM
(C) P can be third. See analysis above for (A).
(D) J can be first. See (B).
(E) Correct. H must be either second or fifth; it cannot be third.

4. K is between J and O, which means the only place for the LH/HL block is after both J and O. Try putting O or J immediately before the LH/HL block, and you will see that M and P must be either first or third (Rule 2), with K in the middle. Can O and J go anywhere else? Sure, J could be first with O fourth, between M and P. In both scenarios, the LH/HL block must be last. The only answer choice that must be true is (D).
(A) K can also be fourth.
(B) O or J can be fifth as well: MJPKOLH
(C) O or J can be first: OKMJPHL
(D) Correct.
(E) J can also be second or fifth.

5. This question seems daunting at first glance, but we can narrow down the choices by consulting previous diagrams.
(A) Our work in question 3 shows that JH can be fourth and fifth or first and second. Since M and P are interchangeable, there are more than two possible arrangements.
(B) From question 2, we know that O and K can be first and second, respectively, but the MP pair and the LH pair are interchangeable, so there must be more than two possible orders.
(C) Correct. Our work from question 4 has the OL block fifth and sixth, respectively. Can it go anywhere else? No. Rules 3 and 4 force us to put J before an OLH block, leaving first and third as the only place for M and P (or P and M). K fills the fourth slot. Since there are only two possibilities, either M is first and P is third or vice versa, this is the correct answer.
(D) The JM block can be either second and third or fourth and fifth. That leaves L and H to square off at the end in any order you like.
(E) This is the same idea as (D). PO can be third and fourth or first and second, leaving LH to spawn additional configurations by swapping slots six and seven.

6. If M is first then P is third. J and O can be second and fifth (any order) or fourth and seventh (respectively).
(A) K can also be fourth if J is second. See question 4.
(B) O can also be second, just before P.
(C) This is correct. If J is second or fifth, the only place for the LH/HL block is sixth and seventh, with K fourth. If J is fourth, LH/HL will occupy slots five and six, while K is stuck with two. Either way, K is before H.
(D) O can be as early as second with L in slots six or seven.
(E) If J is second, K can be fourth, after J.

7. This question has us swap rule 3 for a rule that separates L and H by one space. So now we have three blocks with holes:
J/O_ _O/J
P/M_M/P and
L/H_H/L
One way (the only way, it turns out) to fit these together is in one long chain: LJHMOP. The only restriction is that H must come after J. Let's see how the answer choices compare.
(A) Correct. If O if fourth, J is first, which leaves no room for L.
(B) If K is first, J can be third using the chain from our initial analysis.
(C) See above (B).
(D) A little shuffling shows us this is possible: MJPHOLK
(E) See above (D).
 
m055412
Thanks Received: 0
Forum Guests
 
Posts: 2
Joined: December 30th, 2009
 
 
 

Re: Fortnightly Logic Challenge #23 - Old Blue Eyes

by m055412 Fri Mar 26, 2010 5:05 pm

Here's what I got:

1. A
2. C
3. E
4. D
5. D
6. C
7. A

- Chris
 
katherinewang19
Thanks Received: 0
Forum Guests
 
Posts: 1
Joined: December 21st, 2009
 
 
 

Re: Fortnightly Logic Challenge #23 - Old Blue Eyes

by katherinewang19 Wed Mar 31, 2010 2:27 pm

What I got:

1. A
2. C
3. E
4. D
5. C
6. C
7. A
 
dena.haibi
Thanks Received: 0
Vinny Gambini
Vinny Gambini
 
Posts: 2
Joined: March 24th, 2010
 
 
 

Re: Fortnightly Logic Challenge #23 - Old Blue Eyes

by dena.haibi Fri Apr 09, 2010 3:53 pm

Answers:
1. A
2. C
3. E
4. D
5. C
6. C
7. A

So here is the explanation. Based on the game, you have the following constraints:
1. O _ _ J or J _ _ O
2. M _ P or P _ M
3. HL or LH
4. J-H

Based on rule 4, you know that H cannot audition 1st and J cannot audition last. That provides limited help, but it is something. Certainly the "chunks" will help limit the order of auditions. Time to move to the questions.

1. Which one of the following could be the order of auditions, listed from first to last?

(A) Jordan, Lewin, Hiroshi, Olin, Marin, Kirkwood, Pan
(B) Jordan, Hiroshi, Lewin, Olin, Marin, Pan, Kirkwood
(C) Olin, Hiroshi, Lewin, Jordan, Pan, Kirkwood, Marin
(D) Pan, Olin, Marin, Jordan, Kirkwood, Hiroshi, Lewin
(E) Marin, Jordan, Pan, Lewin, Olin, Hiroshi, Kirkwood

Orientation question. Looking at constraint 1, you see that choice (D) only has one person between O and J, not two, so eliminate it. Constraint 2 prohibits the ordering in choice (B), since there is not one person that auditions between M and P, so eliminate it. Constraint 3 eliminates choice (E), because H and L need to audition back-to-back. You are left with choice (A) and choice (C), and a quick look shows that (C) positions J after H, which violates the final constraint, so the answer is (A).

2. If Olin auditions first, which one of the following must be true?

(A) Marin auditions third.
(B) Pan auditions third.
(C) Kirkwood auditions second.
(D) Jordan auditions third.
(E) Hiroshi auditions seventh.

If O is first, you know that J must be fourth. Since H must come after J, the HL or LH combo must audition fifth and sixth or sixth and seventh. However, if HL or LH auditioned fifth and sixth, that would leave no room for the M _ P or P _ M combo, so HL or LH must audition sixth and seventh. The only spot left is the second position, so K must be second. The audition line then looks like this: O K M/P J P/M H/L L/H.

Alright, so we are looking for something that must be true, so we can eliminate any answer that could be false. (A) is out because M could be third or fifth based on our diagram, so it doesn't have to be true. (B) is out for the same reason, since it is constrained in the same way as M. Looking at (C), you find that in our diagram, K has to be second, so that must be true. (C) is the answer.

On a test, you would move on, but for practice, you can see that (D) is wrong because J cannot be third based on the diagram and (E) is wrong because H could be sixth or seventh.

3. If Lewin auditions immediately before Olin, each of the following could be true EXCEPT:

(A) Kirkwood auditions second.
(B) Marin auditions seventh.
(C) Pan auditions third.
(D) Jordan auditions first.
(E) Hiroshi auditions third.

Another conditional question. The condition is that L comes immediately before O. Due to the HL or LH constraint, you then know that there will be a H L O block. Also, since J must come before H, and it must either be J _ _ O or O _ _ J, you know that there must be a JHLO block. So where can this block go in the order of auditions? One possible scenario would be this: J H L O M/P K P/M. If you try to put J in the second position, you get _ J H L O _ _, which doesn't leave room for the M _ P or P _ M rule, so that order is out. If you try to put J in the third position, once again the M _ P or P _ M rule cannot be satisfied so that is out. If you put J in the fourth position, you get another possible set up: M/P K P/M J H L O.

So the two possibilities are J H L O M/P K P/M OR M/P K P/M J H L O.

For this question, we are looking for the answer that must be false. Choice (A) can be eliminated because K could audition second in the second possible scenario. (B) can be eliminated because M could be seventh in the first scenario. (C) is wrong because P could be third in the second scenario. (D) is wrong because J can be first in the first scenario. That leave (E), which is the correct answer.

Just to verify that (E) is correct, you can see that H cannot be third in either scenario, so that must be false.

4. If Kirkwood auditions sometime between Jordan and Olin, which one of the following must be true?

(A) Kirkwood auditions second.
(B) Either Marin or Pan auditions fifth.
(C) Either Marin or Pan auditions first.
(D) Either Hiroshi or Lewin auditions last.
(E) Jordan auditions either first or fourth.

Another conditional questions, though the condition is a little more open, since it offers four possible orderings of the three elements: J K _ O, O K _ J, J _ K O, or O _ K J. First, let's identify where this group could go if K was in the first space open between J and O. One scenario is the following: J/O K M/P O/J P/M H/L L/H. If we try to move the block over one position, it would be produce _ J/O K _ O/J _ _. There is not room here to satisfy both the M _ P or P _ M and HL or LH conditions, so that set up is out. If you move the block over one space, the same thing happens, so that scenario is out, too. If you move it over one more space, you get _ _ _ J/O K _ O/J. While that leaves room for M _ P or P _ M and HL or LH, there is no ordering that would satisfy the constraint that J comes before H, so this scenario is out.

Now, let's see where the block could go if K were in the second space between J and O. If you put the block first, you would get J/O _ K O/J _ _ _. This crowds out the M and P block and H and L block, so that is not a possibility. If you move it over one position, you get _ J/O _ K O/J _ _. We have found another possible scenario: M/P J/O P/M K O/J H/L L/H. If we move the block over one more, the M and P block and H and L block will be crowded out again, so that can't work. If J/O _ K O/J block occupied positions 4-7, you realize that there is no possible way that J can come before H, so you know that is not a possibility.

We are left with two scenarios, then: J/O K M/P O/J P/M H/L L/H OR M/P J/O P/M K O/J H/L L/H.

Ok, now for the questions. We need to find the choice that must be true. We can see that K does not have to be second in the second scenario, so (A) does not have to be true. (B) can be false in scenario 2 and (C) can be false in scenario 1. Looking at the two scenarios, H or L would have to occupy the last position in both of them, so it looks like (D) is the answer. Time for the next question!

Just for practice, I will check (E). J is either second or fifth in the second scenario, so (E) does not have to be true, and thus is an incorrect answer choice.

5. There are exactly two possible orders of auditions if which one of the following is true?

(A) Hiroshi auditions immediately after Jordan.
(B) Kirkwood auditions immediately after Olin.
(C) Lewin auditions immediately after Olin.
(D) Marin auditions immediately after Jordan.
(E) Pan auditions immediately before Olin.

So this is a little more difficult. I can’t really draw anything out based on the question itself, so I will look at the answer choices.

For choice (A), rather than attempting to write out different possibilities, I look back and see that I have actually already seen this situation in question 3. In that set up, there were two scenarios with H immediately after J, and M and P could be switched in each scenario, so that creates four orderings at least in which H auditions immediately after J. (A) is not the correct answer.

For choice (B), I can look back to my work in question 4. The two scenarios for that question provide more than two exact orderings for the auditions if K auditions immediately after O (OKMJPHL, OKPJMHL, OKMJPLH, OKPJMLH in the first scenario alone). (B) is out.

Looking back at previous work, I could not find any situation that could help me with answer (C). I could move on to see if I can eliminate the other choices based on previous work, but I will just give writing out the constraint a quick try. If L auditions immediately after O, and then H must come immediately after L, due to the constraint, you then know that J must audition three spots before O, because J cannot come after H. The only way that could be set up across seven audition spots is as follows: _ J _ _ O L H. Due to the M _ P or P _ M constraint, you know that M or P must occupy the first or third spots, which means K must then be in the fourth spot. That makes the full ordering to be M/P J P/M K O L. There are only two letters (M and P), that do not have a fixed position, which means there are exactly two orderings of the letters if L auditions immediately after O. (C) is the answer.

If you wanted to show that (D) and (E) are wrong, you can use the work in question 4 again to show that there are definitely more than two exact orderings when M auditions immediately after J and P auditions immediately before O.

6. If Marin auditions first, which one of the following must be true?

(A) Kirkwood auditions before Pan.
(B) Olin auditions after Pan.
(C) Hiroshi auditions after Kirkwood.
(D) Lewin auditions before Olin.
(E) Jordan auditions after Kirkwood.

Ok, so what do we know if M auditions first? We know that P must be third. So the set up must be M _ P _ _ _ _. The O _ _ J or J _ _ O chunk must fit in there somewhere, and given the availability of spots, you realize that the possibilities are either M J/O P _ O/J _ _ OR M _ P J _ _ O. Note that the positions of O and J are fixed in the second scenario because J cannot be last because it has to come before H. If you add in the HL or LH constraint, you finally get these two possibilities M J/O P K O/J H/L L/H OR M K P J H/L L/H O.

Now on to the answer choices. We need to find what must be true. In the first scenario, K auditions after P, so (A) is incorrect. Also, in scenario one, O could audition before P, so you can eliminate (B). In both scenarios, though, H does audition after K, so (C) is the answer that must be true.

If you aren’t comfortable that you have thought of all the scenarios, you can compare (D) and (E) to the two scenarios we have. L can audition after O in scenario one, and J can audition before K in scenario one, so neither (D) nor (E) must be true. That would leave only choice (C) as an answer with a condition that had not been shown to be false in a given scenario.

7. If the condition that Hiroshi’s audition will take place immediately before or immediately after Lewin’s audition is replaced with the condition that exactly one audition takes place between Hiroshi and Lewin, and if all other conditions remain in effect, each of the following could be true EXCEPT:

(A) Olin auditions fourth.
(B) Jordan auditions third.
(C) Kirkwood auditions first.
(D) Lewin auditions sixth.
(E) Hiroshi auditions fourth.

This question changes a constraint (HL or LH changes to H _ L or L _ H). With seven available spots, given the constraints on J and O and M and P combined with the new constraint for H and L, you realize that there must be an open space on both sides of both J and O. Otherwise, all the pieces could not fit within the seven spots. So that means that J/O _ _ O/J _ _ _ and _ _ _ J _ _ O are not possible scenarios, since both M _ P or P _ M and H _ L or L _ H could not fit. The leaves the possibilities as _ J/O _ _ O/J _ _ OR _ _ J/O _ _ O/J _. In scenario one, K would have to be seventh to fit the other constraints, and in scenario two K would have to be first to fit the other two constraints. We don’t know whether the M and L or the H and L block comes first in either scenario, since the only additional limiting factor is that J must become H. So the most we can determine is the following two set ups: _ J/O _ _ O/J _ K or K _ J/O _ _O/J _.

This question means that we are looking for an answer that must be false. If we compare (A) to our scenarios, we see that O cannot be fourth in either scenario. This must be false, so (A) is our answer.

To show the other answers are wrong, we can compare the answers to the scenarios to show that each could be true in at least one case. J can audition third in scenario two, so (B) is wrong. K can audition first in scenario two so (C) is wrong. L could audition sixth in scenario one as long as J was in position two, so (D) is wrong. H could audition fourth in either scenario, as long as J was in the earlier position, so (E) is wrong.

Now, if only I could get through all that on the first try in 8 minutes...
 
anthony.t.dinoto
Thanks Received: 0
Forum Guests
 
Posts: 2
Joined: April 18th, 2010
 
 
 

Re: Fortnightly Logic Challenge #23 - Old Blue Eyes

by anthony.t.dinoto Thu Apr 22, 2010 2:31 pm

1- a
2- c
3- e
4- d
5- c
6- c
7- a
 
jaydizzle
Thanks Received: 0
Vinny Gambini
Vinny Gambini
 
Posts: 14
Joined: January 10th, 2010
 
 
 

Re: Fortnightly Logic Challenge #23 - Old Blue Eyes

by jaydizzle Tue May 25, 2010 7:40 pm

Any official PDF answer and explanation to this one? I had some trouble with this one.
User avatar
 
noah
Thanks Received: 1192
Atticus Finch
Atticus Finch
 
Posts: 1541
Joined: February 11th, 2009
 
 
 

Re: Fortnightly Logic Challenge #23 - Old Blue Eyes

by noah Wed May 26, 2010 9:50 am

jaydizzle Wrote:Any official PDF answer and explanation to this one? I had some trouble with this one.

I'm afraid we're busy explaining actual LSAT questions! (not to dis our own too badly!). But there are some explanations above. See if those help.